Đến nội dung

Karl Heinrich Marx nội dung

Có 54 mục bởi Karl Heinrich Marx (Tìm giới hạn từ 26-04-2020)



Sắp theo                Sắp xếp  

#705028 VN TST 2018

Đã gửi bởi Karl Heinrich Marx on 06-04-2018 - 02:13 trong Thi HSG Quốc gia và Quốc tế

Bảng này không phải bảng đầy đủ nhé bạn, lấy dãy nhị phân giống 2016 cột đầu của hai hàng này và đằng sau cho hai số 1,1 là thấy rõ.

À xin lỗi mình không chú ý đến điều kiện là đầy đủ. Nếu như vậy thì chỉ cần thêm một chút lập luận là xét một chuỗi $A$ bất kì sinh ra từ cái dòng ta muốn xóa. Nếu dòng này có một số $a$ nào đó là $0$ hoặc $1$ nằm ngoài $k$ cột đã nêu thì xét một chuỗi giống y chuỗi $A$ chỉ là thay cái cột chứa $a$ bằng $1-a$ (tức là đảo $0$ với $1$ và $1$ với $0$ thôi) thì bảng đầy đủ nên tồn tại một hàng sinh ra được chuỗi mới này và dĩ nhiên ở cái cột đó thì hàng đó không chứa gì cả, vì cột này chứa $a$ thì không chứa $1-a$. Do đó hàng này sẽ sinh ra được chuỗi $A$.




#704841 VN TST 2018

Đã gửi bởi Karl Heinrich Marx on 04-04-2018 - 03:15 trong Thi HSG Quốc gia và Quốc tế

Thực ra lúc mình thi thì mình nhớ rằng ý b) không có điều kiện "tất cả các cột còn lại đều trống". Thực ra điều kiện chỉ cần có đúng $k$ cột có chứa hai số $0,1$ là đủ rồi. Lời giải của bạn vẫn đúng về ý tưởng nhưng có một xíu cải tiến như sau. Vẫn xét hàng mà chúng ta có thể xóa được trong lập luận của bạn, nếu nó không có phần tử nào ở các cột còn lại thì có thể xóa nó đi. Nhưng nếu nó có, và giả sử là số $0$ chẳng hạn ở một cột nào đó ngoài $k$ cột này thì ta vẫn có thể chứng minh mọi dãy nhị phân sinh ra từ hàng này thì có thể sinh ra từ một hàng khác bởi nếu không ta có thể lập luận và tạo thêm một cột nữa có chứa cả hai số $0,1$ và ta có điều vô lý.

Giả sử chọn $k = 2016$, và có $2^{2016}+1$ dòng, trong số các dòng này chứa tất cả các chuỗi nhị phân độ dài $k$ và có đúng 2 chuỗi giống nhau, xét hai chuỗi giống nhau ở cột thứ $2017$, một chuỗi chứa 0 một chuỗi để trống, ở cột thứ $2018$ thì chuỗi để trống ở cột $2017$ chứa 0 còn chuỗi còn lại thì để trống. Khi đó thì không thể xóa đi dòng nào được đâu bạn ạ.

 

Bài 5 ý b mình không tìm được một cách lập luận nào chặt chẽ và hợp lí cả cho trường hợp mà một số chẵn và một số lẻ và số lẻ lớn hơn số chẵn. Bạn có ý tưởng gì không?




#704769 VN TST 2018

Đã gửi bởi Karl Heinrich Marx on 03-04-2018 - 01:06 trong Thi HSG Quốc gia và Quốc tế

Bài 2:

Câu b đã là gợi ý cho câu a. Một bảng gồm $2^k$ xâu nhị phân độ dài là $k$ và phần còn lại chừa trống thỏa mãn điều kiện là bảng tối giản.

câu b ta sẽ chứng minh nếu $m>2^k$ thì ta sẽ xóa được một dòng sao cho bảng vẫn còn là tối giản. Điều này có nghĩa là chứng minh rằng tất cả chuỗi nhị phân sinh ra từ dòng bị xóa sẽ thu được bằng cách thêm vào $0,1$ ở các dòng khác. Ta chỉ quan tâm tới cách sinh ra các chuỗi nhị phân với $k$ cột chứ $0,1$. Bây giờ xét tập A gồm $2^k$ chuỗi nhị phân có độ dài $k$. Mỗi một bước ta chọn một dòng của bảng (chỉ $k$ cột chứa 0,1), và xóa đi tất cả những chuỗi nhị phân trong A có thể sinh ra được bằng cách thêm $0,1$ vào các ô còn trống của dòng này. Với một dòng mà ta không xóa được trong A bất kì một chuỗi nào thì có nghĩa tất cả các chuỗi nhị phân sinh ra từ dòng đó có thể được sinh ra bởi các dòng khác, tức là xóa đi dòng đó bảng vẫn tối giản. A chỉ có $2^k$ phần tử mà ta có $m>2^k$ dòng thì dĩ nhiên sẽ chọn được một dòng chẳng xóa đi phần tử nào trong tập A được.

Bài 3:

Ta có một nhận xét là $(k,n)=1$ thì $(n-k,n)=1$, tức là ta có thể chia tập $A_n$ thành 2 tập $B_n$ và $C_n$, sao cho tập $B_n$ chứa tất cả các phần tử của $A_n$ mà bé hơn $\frac{n}{2}$, $C_n$ chứa các phần tử còn lại. Khi đó ta có $ k \in B_n \Rightarrow n-k \in C_n$.

Đặt $c_n$ là phần tử nhỏ nhất của $C_n$ thì $c_n> \frac{n}{2}>1$, chú ý là $B_n$ luôn chứa $1$ và $C_n$ luôn chứa $n-1$. Mình quên latex nhiêu rồi nên lười viết ra nhưng có thể dễ dàng suy ra là $P_n(x)=(1+x^{c_n-1}).Q_n(x)$. Vậy luôn tồn tại $r_n \ge 1$ thỏa mãn.

ý b thì để $P_n(x)$ bất khả quy thì $Q_n(x)=1$, điều này có nghĩa là $A_n$ có đúng 2 phần tử suy ra $P_n(x)=1+x^{n-2}$ và để nó bất khả quy trên Z thì phải tồn tại $m$ để $n-2 = 2^m$, đến đây thì ta cần tìm $n$ dạng $2^m+2$ mà $(n,k)>1$ với mọi $1<k<n-1$, nếu $m>2$ thì suy ra $(3,n)>1$ suy ra $3 | 2(2^{m-1}+1)$ suy ra $m$ chẵn. Mà $(5,n)>1$ nên cũng suy ra $5 | 2(2^{m-1}+1)$  suy ra $m$ lẻ. Mâu thuẫn!

Vậy $m \le 2$, tìm được $3$,$4$ và $6$ thỏa đề bài.




#623181 Việt Nam TST 2016 - Thảo luận đề thi

Đã gửi bởi Karl Heinrich Marx on 28-03-2016 - 17:53 trong Thi HSG Quốc gia và Quốc tế

Một ý tưởng cho bài 6, tính toán chắc hơi trâu bò tí.

Đầu tiên gọi một đa thức $P$ là tốt nếu $V(P)=0$, ta có thể thấy là với $a$ là một hằng số thực và $P,Q$ là hai đa thức tốt thì $P+aQ$ cũng tốt.

Từ điều này ta có thể phát biểu thế này:

$PQ$ tốt với mọi $P$ là đa thức có bậc bé hơn $8$ khi và chỉ khi $Q,xQ,x^2Q,...,x^7Q$ đều là các đa thức tốt.

Bây giờ đặt $Q(x)= x^8+\sum_{i=0}^{7} a_ix^i$

Thay vào điều kiện $8$ đa thức tốt trên thì ta sẽ thu được một hệ $8$ phương trình với $8$ ẩn là các số $a_i$. Hệ này nếu có nghiệm thì phải có nghiệm duy nhất (vì nó tồn tại nên phải cm đc là nó có nghiệm). Thông thường để chặt chẽ chắc là dùng ma trận (thực ra chém thế chứ mình k biết dùng ma trận ntn :v), tuy nhiên trong phạm vi toán phổ thông thì chắc phải mò nghiệm, bằng cách phải thử những trường hợp đơn giản, ở đây ta thấy số 8 ko quá quan trọng nên có thể thay 8 bởi 1,2,3 gì đó để đoán xem hệ ntn, nghiệm sẽ ntn rồi tổng quát lên cho 8.  Có lẽ phải dựa vào kq của bước này mới chứng minh là $Q$ có $8$ nghiệm phân biệt.




#568123 Tồn tại vô hạn $n$ để $d(n)$ không chia hết $d(a^2+b...

Đã gửi bởi Karl Heinrich Marx on 25-06-2015 - 16:40 trong Số học

(Cách khác)

Lời giải :

Ta chọn $n$ như sau :

$$n=2^{p-1}p_2p_3...p_k$$

Trong đó $p$ là số nguyên tố, và $p_2,p_3,...,p_k$ là $k$ số nguyên tố phân biệt lớn hơn $3$.

Hiển nhiên có vô số số $n$ như vậy, và hiển nhiên rằng $\omega (n)=k$.

Ta sẽ chứng minh với cách này thì với mọi cặp $(a,b)$ nguyên dương có tổng bằng $n$ thì ta đều có $d(n)\nmid d(a^2+b^2)$.

Thật vậy, giả sử tồn tại một cặp số nguyên dương $(a,b)$ thoả $a+b=n$ và $d(n)\mid d(a^2+b^2)$.

Dễ thấy $d(n)=2^{k-1}.p$. Suy ra :

$$p\mid d(a^2+b^2)\Rightarrow q^{p-1}\mid a^2+b^2$$

với $q$ là một ước nguyên tố nào đó của $a^2+b^2$. Thế thì :

$$q^{p-1}\leq a^2+b^2< (a+b)^2=4^{p}p_2^2p_3^2...p_k^2$$

Rõ ràng nếu $q<4$ thì với $p$ đủ lớn ta sẽ có $q^{p-1}>4^{p-1}p_2^2p_3^2...p_k^2$. Như vậy $q=2,3$.

Nếu $q=3$ thì suy ra $3\mid a^2+b^2$, suy ra $3\mid a,b$. Kéo theo $3\mid n$.

Dễ thấy điều này mâu thuẫn với cách chọn $n$ như trên.

Vậy phải có $q=2$. Dẫn tới :

$$2^{p-1}\mid a^2+b^2$$

Rõ ràng $a,b$ cùng tính chẵn lẻ, nhưng chúng không thể cùng lẻ vì khi đó $a^2+b^2\equiv 2\pmod 4$, vậy phải có $a,b$ cùng chẵn.

Đặt $a=2^A.x,b=2^B.y$ ($x,y$ lẻ), không giảm tổng quát có thể giả sử $A\geq B$.

Ta có :

$$2^{p-1}\mid 2^{2A}x^2+2^{2B}y^2=2^{2B}(2^{2A-2B}x^2+y^2)$$

Chú ý rằng $2^{2A-2B}x^2+y^2\equiv 1,2,3\pmod 4$. Do đó suy ra :

$$2B+1\geq p-1\Rightarrow A\geq B\geq \dfrac{p-1}{2}$$

Suy ra rằng $2^{(p-1)/2}\mid a,b$

Như vậy ta có thể có được biểu diễn sau :

$$a^2+b^2=2^{p-1}(a_1^2+b_1^2)$$

Do $d(n)$ là hàm nhân tính nên :

$$p\mid d(a^2+b^2)=d(2^{p-1}.(a_1^2+b_1^2))=p+d(a_1^2+b_1^2)\Rightarrow p\mid d(a_1^2+b_1^2)$$

Tương tự trên ta được :

$$2^{\frac{p-1}{2}}\mid a_1,b_1$$

Tiếp tục quá trình này, ta sẽ suy ra :

$$2^{\frac{p-1}{2}.N}\mid a,b$$

Với số nguyên dương $N$ tuỳ ý, rõ ràng điều này là vô lí.

Ta có điều cần chứng minh.

Sorry anh hiểu nhầm là $d(a^2+b^2)|d(n)$ thay vì $d(n)|d(a^2+b^2)$, có thể xử lí đoạn sau phần $2^{p-1}|a^2+b^2$ như thế này:

Nếu $v_2(a)>v_2(b) \Rightarrow p-1=v_2(n)=v_2(a+b)=v_2(b) \Rightarrow v_2(a^2+b^2)=2v_2(b)=2p-2 \Rightarrow p \nmid d(a^2+b^2)$

Nếu $v_2(a)=v_2(b)$ thì có thể thấy $v_2(a+b) \ge v_2(a)+1 \Rightarrow v_2(a) \le p-2$. Ngoài ra dĩ nhiên ta có $v_2(a^2+b^2)=2v_2+1<2p-1$ như vậy để $p|d(a^2+b^2$ ta phải có $2v_2(a)+1=p-1$ điều này mâu thuẫn vì một bên chẵn, một bên lẻ.

Thử xem nếu bài toán này mà thay điều kiện $d(n) \nmid d(a^2+b^2)$ thành $d(n) \nmid d(2a^2+2b^2)$ thì phải xử lí thế nào và liệu bài toán còn đúng không?

Ta thử xem với những giá trị $k$ nào (không nhất thiết tổng quát chỉ cần những ước lượng nào đó với $k$) thì thay điều kiện thành $d(n) \nmid d(a^2+kb^2)$ bài toán vẫn đúng.




#568024 Tồn tại vô hạn $n$ để $d(n)$ không chia hết $d(a^2+b...

Đã gửi bởi Karl Heinrich Marx on 25-06-2015 - 08:14 trong Số học

Chứng minh rằng với mọi số nguyên dương $k$ luôn tồn tại vô hạn số nguyên dương $n$ sao cho $n$ có đúng $k$ ước nguyên tố và thoả mãn $d(n)$ không chia hết $d(a^2+b^2)$ với mọi cặp số nguyên dương $(a,b)$ thoả mãn $a+b=n$.

Trong đó kí hiệu $d(n)$ là số các ước dương của $n$.

Trước tiên ta nhận xét là chỉ có SCP mới có số ước là lẻ thôi. Vì vậy ta nghĩ đến việc sẽ chọn $n$ là SCP và tìm $n$ sao cho với $a+b=n$ thì $a^2+b^2$ không là SCP. Nếu $a^2+b^2$ là SCP thì phải tồn tại $x,y$ để $a=x^2-y^2,b=2xy$ như vậy $n=x^2-y^2+2xy=(x+y)^2-2xy$

Tuy nhiên đến đây ta thấy là với mọi $n$ chính phương thì phương trình $n=u^2-2v^2$ luôn có nghiệm.

Nhưng không sao ta vẫn có thể có 1 giải pháp khác. Ta chọn $n$ là một SCP mà $k$ ước nguyên tố của $n$ đều có dạng $8t+2$ hoặc $8t+5$, khi đó $2$ không là thặng dư chính phương của những số nguyên tố này nên nếu $u^2-2v^2=n$ thì cả $u$ lần $v$ đều phải chia hết cho tất cả những số nguyên tố này Do vậy dễ thấy $\sqrt{n}$ phải là ước của cả $x+y$ và $y$ suy ra $(x,y)>\sqrt{n}$ như vậy $a^2+b^2=(x^2+y^2)^2 \vdots n^2$ nên dĩ nhiên là $d(n)$ không chia hết cho $d(a^2+b^2)$. Còn nếu $a^2+b^2$ không là SCP thì $2|d(a^2+b^2)$ là $d(n)$ là số lẻ nên cũng không chia hết.

Trong lời giải này vướng mắc một chút ở chỗ với $k$ đủ lớn phải chứng minh tồn tại các số nguyên tố dạng $8t+2$ hoặc $8t+5$. Có nghĩa là phải cm có vô hạn những số nguyên tố dạng này.

Thực ra có một định lí là tồn tại vô hạn số nguyên tố dạng $ak+b$  với $(a,b)=1$ nhưng chứng minh vô cùng khó. Có một đl khác cm đơn giản hơn nhiều là tồn tại vô hạn số nguyên tố dạng $pk+1$. Tuy nhiên cái này k dùng vào bài này. Đây chỉ là một hướng khả thi, có lẽ là có 1 cách tiếp cận khác tốt hơn để tránh vấn đề này.




#564652 Tìm số tập con của tập $S=\begin{Bmatrix} 1;2;...;n...

Đã gửi bởi Karl Heinrich Marx on 09-06-2015 - 19:34 trong Tổ hợp và rời rạc

Có 2 ý tưởng để đếm bài này, đầu tiên dễ thấy là truy hồi. Gọi $S_n$ là tập hợp các tập con thỏa mãn với $n$ thì một tập mà thuộc $S_{n+1}$ nó sẽ là một tập trong $S_n$ còn nếu không thì nó chứa $n+1$ suy ra các phần tử của nó bé hơn hoặc bằng $n-2$ mà thỏa mãn đk bài toán. Vậy $|S_{n+1}|=|S_{n-2}|+|S_n|$.

Ý tưởng thứ hai là lập một song ánh để cố phá cái điều kiện hơn kém ít nhất 3 đơn vị đi. Bây giờ giả sử một tập hợp thỏa mãn đề bài mà có $i$ phần tử là $a_1<a_2<...<a_i$ khi đấy nếu ta chuyển sang cái tập này ${b_j=2(i-j)+a_j, 1 \le j \le i}$ thì đây là một tập con bình thường của tập $S$ mà trong đó các phần tử của nó đều không nhỏ hơn $2i-1$. Ngược lại từ một tập con của $n$ có $i$ phần tử thỏa mãn $b_1<b_2<..<b_i$ và $b_1 \ge 2i-1$ khi đó lật lại ta có tập ${a_j=b_j-2(i-j), 1 \le j \le i}$ đây là một tập con thỏa mãn đề bài, vậy số tập con có $i$ phần tử thỏa mãn đề bài là ${n-2i+2\choose i}$. Cho $i$ chạy từ $0$ đến $n$ tính ra kết quả.

Kể ra ta thu được một đẳng thức tổ hợp bằng 2 cách đếm.




#564726 Tìm số tập con của tập $S=\begin{Bmatrix} 1;2;...;n...

Đã gửi bởi Karl Heinrich Marx on 10-06-2015 - 00:03 trong Tổ hợp và rời rạc

Em cứ nghĩ là sẽ tính được để cho thêm thầy Thanh một đẳng thức vào bộ sưu tập chứ :D




#566327 Tìm nghiệm nguyên

Đã gửi bởi Karl Heinrich Marx on 17-06-2015 - 05:39 trong Số học

Tìm nghiệm nguyên của phương trình:

(x+y2)(x2+y)=(y-x)2

Trước tiên đặt $d>0$ là UCLN của $x$ và $y$ trong đó $x=da,y=db \Rightarrow (a,b)=1$ như vậy pt trên tương đương với:

$$(a+db^2)(b+da^2)=(a-b)^2 (*)$$

Không mất tính tổng quát ta giả sử $|a| \ge |b|$, từ biểu thức trên ta suy ra:

$$4a^2 \ge (a-b)^2 \ge da^2+b \ge 0 \Rightarrow d \le 5$$

Nếu mà $d=5$ thì những đẳng thức trên xảy ra đồng thời và ta có $a=-b$

Khi đó thay vào $(*)$ ta được:

$$(5b-1)(5b+1)=4 \Rightarrow 5b^2=1$$

loại nghiệm này.

Vậy $1 \le d \le 4$

Ngoài ra ta biến đổi biểu thức ban đầu theo một cách khác:

$(x+y^2)(y+x^2)=(y-x)^2 \Leftrightarrow xy(xy+2)=(1-x-y)(x^2-xy+y^2) \Leftrightarrow ab(d^2ab+2)=(1-da-db)(a^2-ab+b^2)$

Vì $a,b$ nguyên tố cùng nhau nên biểu thức $a^2-ab+b^2$ luôn là số lẻ. Do đó nếu $d$ chẵn thì bên trái chia hết cho 2 mà bên phải thì không. Vậy $d$ phải lẻ.

Ngoài ra cũng từ biến đổi cuối cùng ở trên ta suy ra được $(a^2-ab+b^2)|d^2ab+2$.

Nếu $d=1$ thì ta suy ra $ab+2 \ge a^2-ab+b^2 \Rightarrow 2 \ge (a-b)^2$

Mà $a$ không thể bằng $b$ nên ta suy ra $|a-b|=1$

Đến đây thay vào giải khá đơn giản ra $a=1,b=0$ và ngược lại.

Bây giờ nếu $d=3$ cũng từ biểu thức trên ta suy ra $ab|(1-da-db) \Rightarrow a|1-3b$

Đặt $|1-3b|=k|a|$ thì ta chặn được $1 \le k \le 2$

Đến đây thì còn vài trường hợp thay vào và giải ra đáp án, mình không rõ là có cách nào để loại bớt trường hợp nữa không nhưng đến đây chắc khả thi rồi.




#718995 Trên đường tròn cho n điểm

Đã gửi bởi Karl Heinrich Marx on 03-01-2019 - 02:08 trong Tổ hợp và rời rạc

Trên đường tròn cho n điểm, từ các điểm nói trên hiển nhiên ta có ${C_n}^2$ đoạn thẳng và tổng số các đa giác là

$({C_n}^3 + {C_n}^4 + {C_n}^5 + ... + {C_n}^n)$ .Hãy tìm số cách bỏ bi các đoạn thẳng sao cho không còn đa giác nào , biết rằng mỗi cách bỏ đi các đoạn thẳng như vậy thì mỗi điểm nói trên , vẫn tồn tại ít nhất một đoạn thẳng nhận nó làm đầu mút ?

Yêu cầu bài này phải phát biểu lại là với một cấu hình xóa đi các đoạn thẳng thỏa mãn 2 điều kiện nêu ra thì cấu hình này có nhiều nhất bao nhiêu đoạn thẳng.

 

Có nhiều nhất là $n-1$ đoạn thẳng (vd $1$ điểm nối với $n-1$ điểm còn lại chẳng hạn). Cái này liên quan đến định nghĩa cây ( đồ thị liên thông, không có chu trình) trong lý thuyết đồ thị.

 

Mình sẽ chỉ ra một cách chứng mình mình nghĩ ra (tham khảo sách về tính chất của cây với $n$ điểm có thể bạn sẽ tìm thấy chứng minh hay hơn).

 

- Hai điểm gọi là liên thông nếu có một đường đi tạo bởi ít nhất 2 đoạn thẳng nối hai điểm đó.

 

- Nếu một cấu hình thỏa mãn 2 điều kiện bài toán mà có hai điểm không liên thông, ta có thể nối chúng lại với nhau để có một cấu hình mới thỏa mãn yêu cầu mà có số đoạn thẳng lớn hơn. Do đó cần tìm max số đoạn thẳng trong trường hợp 2 điểm bất kì trong $n$ điểm liên thông với nhau.

 

- Xét một cấu hình thỏa mãn : không có đa giác, mỗi điểm trong $n$ điểm là mút của ít nhất một đoạn thẳng, cấu hình này có nhiều đoạn thẳng nhất và 2 điểm bất kì (khác 2 mút một đoạn thẳng) trong $n$ điểm liên thông với nhau. Xét tập $S$ chứa một điểm bất kì trong $n$ điểm.

 

- Mỗi bước ta xóa đi một đoạn thẳng thỏa mãn : đoạn thẳng này có một mút là một điểm thuộc $S$ và một điểm không thuộc $S$ và sau đó thêm cái điểm nằm ngoài $S$ của đoạn thẳng vừa xóa vào $S$.

 

- Rõ ràng khi tập $S$ chưa lấp đầy bởi $n$ phần tử thì luôn tìm được ít nhất một đoạn thẳng thỏa mãn để xóa. Nếu không các điểm trong $S$ không liên thông với điểm nào ngoài $S$.

 

- Khi đó tất cả các điểm thuộc $S$ mà không nối với nhau bởi một đoạn thẳng đã bị xóa thì liên thông với nhau.

 

- Khi tập $S$ đầy $n$ phần tử thì không còn đoạn thẳng nào nữa. Vì nếu tồn tại một đoạn thẳng nối 2 điểm trong $S$ mà đoạn này chưa bị xóa, khi đó tồn tại một đường đi lớn hơn 2 đoạn thẳng nối 2 điểm đó, như thế thì cấu hình ban đầu chứa một đa giác.

 

Vậy cấu hình ban đầu nêu ra có đúng $n-1$ đoạn thẳng.




#675485 Một bài tổ hợp từ một bài số học

Đã gửi bởi Karl Heinrich Marx on 27-03-2017 - 23:23 trong Tổ hợp và rời rạc

định nghĩa một xích là $1$ dãy các tập hợp $(A_{a_1};A_{a_2};...;A_{a_t})$ thoả mãn $0<a_1<a_2<...<a_t<T+1$ và $A_{a_i}\subset A_{a_{i+1}} \forall 1\leq i\leq t-1$. Với mỗi tập $A_i$, gọi $f(A_i)$ là xích dài nhất nhận $A_i$ là tập đầu tiên trong xích. Giả sử mọi xích đều có độ dài $\leq n-1$ thì lúc đó tồn tại $n$ tập $A_{b_i} \forall i=\overline{1,n}$ thoả mãn $f(A_{b_i})=f(A_{b_j}) \forall i,j=\overline{1,n}$. Nếu tồn tại $i;j$: $1\leq i< j\leq n$ để $A_{b_i}\subset A_{b_j}$ thì $f(A_{b_i})\geq f(A_{b_j})+1$ (vô lí). Vì vậy $n$ tập trên không thoả mãn đề bài, suy ra giả sử sai. Vậy tồn tại $1$ xích độ dài $n$, nhận $A_t$ làm tập cuối cùng trong dãy. Lúc đó $\left | A_t \right |\geq n\Rightarrow A_t=A$.

Lời giải của bạn chính xác rồi, lời giải của mình hơi khác một tí nhưng chung quy vẫn là nguyên lí Dirichlet, thế theo bạn bài toán này phát biểu một cách số học thì nó như thế nào?




#675400 Một bài tổ hợp từ một bài số học

Đã gửi bởi Karl Heinrich Marx on 26-03-2017 - 20:42 trong Tổ hợp và rời rạc

Thể theo nguyện vọng của đồng chí supermember, lâu lâu lên diễn đàn trao đổi với đồng chí một tí, cũng hi vọng là có thể chia sẻ trao đổi chút gì đó với các bạn.

Từ một bài số học mình biết cũng khá lâu, hôm nay rỗi rãi phát biểu nó sang một bài tổ hợp. Cho một tập hợp $A$ gồm $n$ phần tử, và một dãy tập hợp con khác rỗng phân biệt của $A$ là $A_1,A_2,...,A_T$ với $T=(n-1)^2+1$. Với một dãy số nguyên dương tăng bất kì $a_1,a_2,..,a_n$ mà $a_n \le T$ thì tồn tại $i,j,1 \le i < j \le n$ và $A_{a_i}$ là một tập con của $A_{a_j}$. Chứng minh cái dãy tập hợp trên chứa A.

Thử giải xem nào đại ca supermember. Qua chủ đề này có thể mình sẽ nói 1 chút về sự liên hệ giữa các con số và tập hợp. Theo các bạn là nó có liên hệ gì không?




#585101 HỌP MẶT LẦN 2 NĂM 2015 TẠI TP HỒ CHÍ MINH

Đã gửi bởi Karl Heinrich Marx on 26-08-2015 - 18:41 trong Góc giao lưu

 

-          Nhan sắc VMF (trên VMF có cô nào xinh không =]] )

 

Tình hình là tên đồng chí có xuất hiện trong list nhân sự VMEO nhưng mà chưa đóng góp được gì, đề nghị đồng chí làm tốt nhiệm vụ in đậm ở trên, thu thập thông tin lẫn fb của các em xinh tươi, đồng chí hoạt động ở box nào thì gửi thông tin về trưởng ban chọn đề box đấy để kiểm duyệt =)) có gì chúng ta sẽ trao đổi thêm về vấn đề này sau buổi offline (vừa gửi lời mời kết bạn trên fb rồi). 




#584795 CMR: tồn tại một chỉ số $i_o$ sao cho $a_{i_0}\...

Đã gửi bởi Karl Heinrich Marx on 25-08-2015 - 01:11 trong Số học

Bài toán: Cho tập $S=\{a_1, a_2, ..., a_n\}\subset \mathbb{N^*}$ và $P(x)\in\mathbb{Z}[x]$. Giả sử rằng với mọi $k\in\mathbb{Z^+}$ đều tồn tại chỉ số $i$ sao cho $a_i\;|\;P(k)$. CMR: tồn tại một chỉ số $i_o$ sao cho $a_{i_0}\;|\;P(k), k\in\mathbb{Z}$$

Trước tiên đặt $a$ là BCNN của tất cả các phần tử thuộc $S$.

Với mọi số $r \in Z$ thì luôn tồn tại số $k$ để $r+ka \in Z^+$, khi đó thì $P(r) \equiv P(r+ka)$ (mod $a$), mặt khác tồn tại $a_i$ để $a_i|P(r+ka)$ và $a_i|a$ ta suy ra $a_i|P(r)$.




#566081 CMR: Trong 100 số tự nhiên đó, tồn tại 2 số bằng nhau.

Đã gửi bởi Karl Heinrich Marx on 16-06-2015 - 01:31 trong Bất đẳng thức và cực trị

Gợi ý:

Gợi ý:

Gợi ý:

Haha dĩ nhiên là tìm được chứ thầy, vd lấy 90 số 25 và 10 số 100 thì có tổng bằng 19 rồi :v

Nhưng mà em hiểu ý của thầy, bài toán này câu hỏi đặt ra chỉ là vì mục đích che giấu cái bđt thầy đã nêu ra để đánh đố người khác. Cái này phù hợp vs thi cử :D

Tuy nhiên ta thử đặt ra những vấn đề mới đi vào trọng tâm xem thế nào.

Ví dụ thế này, tìm tất cả số tự nhiên $n$ có thể biểu diễn được dưới dạng

$$\frac{1}{\sqrt{a_1}}+\frac{1}{\sqrt{a_2}}+...+\frac{1}{\sqrt{a_{100}}}$$

trong đó $a_1,a_2,..,a_{100}$ là các số tự nhiên phân biệt.

Hoặc là tìm tất cả các số tự nhiên $n$ mà tồn tại $k$ tự nhiên để

$$n=\frac{1}{\sqrt{a_1}}+\frac{1}{\sqrt{a_2}}+...+\frac{1}{\sqrt{a_k}}$$

trong đó $a_1,a_2,..,a_{k}$ là các số tự nhiên phân biệt.

Khó hơn một chút thì là với mỗi số nguyên dương $t$, hỏi có những số tự nhiên $n$ nào mà thỏa mãn tồn tại $k$ để 

$$\frac{n}{t}=\frac{1}{\sqrt{a_1}}+\frac{1}{\sqrt{a_2}}+...+\frac{1}{\sqrt{a_k}}$$

trong đó $a_1,a_2,..,a_{k}$ là các số tự nhiên phân biệt.

Khó hơn chút nữa thì tìm những số tự nhiên $n$ mà tồn tại $k$ để:

$$\frac{n}{k}=\frac{1}{\sqrt{a_1}}+\frac{1}{\sqrt{a_2}}+...+\frac{1}{\sqrt{a_k}}$$

trong đó $a_1,a_2,..,a_{k}$ là các số tự nhiên phân biệt.

Trong mỗi trường hợp tồn tại trên ta thử tìm số $k$ nhỏ nhất để có thể biểu diễn được xem thế nào.

Hoặc ta cố định $k$, cho trước hai số nguyên dương $k$ và $t$ thì những $n$ nào thỏa mãn:

$$\frac{n}{t}=\frac{1}{\sqrt{a_1}}+\frac{1}{\sqrt{a_2}}+...+\frac{1}{\sqrt{a_k}}$$

trong đó $a_1,a_2,..,a_{k}$ là các số tự nhiên không nhất thiết phân biệt.

 

Haha nếu thầy thấy bài trên có vẻ không thực tế thì giải quyết những vấn đề em nêu trên nhé :D




#564981 CMR: $\sum_{k=0}^n (-1)^k{m-1\choose k}{m\choose n-k}=(-1...

Đã gửi bởi Karl Heinrich Marx on 11-06-2015 - 18:41 trong Tổ hợp và rời rạc

Phải nói rằng, bao nhiêu kỹ thuật phân tách, mẹo mực để sáng tạo ra được một bài toán đã bị một chiêu biến hoá kỳ ảo của em giải quyết ngon lành. Chiêu này em học được ở đâu vậy? Có thời gian mong em viết một bài hướng dẫn cách sử dụng và phạm vi ứng dụng của nó được không?

Ngày xưa có một ông anh khóa trên chỉ cho em cái chiêu này và nhất là ông này đc đi IMO về nên trong lúc ôn có nhiều bài tập tính mấy tổng tổ hợp này, trong quá trình giải thì e cũng tìm ra một số kĩ thuật mới kiểu như thay đổi cận của sigma như bài trên. Thầy và các bạn có thể tham khảo về bài viết về hàm sinh trong tài liệu này (chuyên đề tổ hợp của MS):

 http://diendantoanho...-của-mathscope/

Thực ra nó còn một kĩ thuật nữa mà trong tài liệu có đề cập là giả sử với một tổng tổ hợp có một thành phần cố định là $m$ chẳng hạn (vd như $m$ hoặc $n$ ở bài trên) thì ta gắn thêm cho biểu thức một đại lượng $x^m$ sau đó cho $m$ chạy từ $0$ đến $\infty$ sẽ được một hàm sinh, dùng chuyển dấu sigma sẽ linh hoạt trong biến đổi hơn và có một số kĩ thuật nhỏ khác trong quá trình xử lí.

Thực tế thì ngày xưa lúc đấy chỉ chăm chăm vào giải toán nên kĩ thuật mới nghĩ ra cũng chỉ nhất thời để giải quyết bài toán đang làm chứ không chiêm nghiệm, phát triển nó thêm còn giờ thì không còn đủ hăng say để tiếp tục nữa và cũng quên nhiều rồi. Hehe thầy Thanh nếu có thể thì tham khảo và nghiên cứu đưa ra cái gì mới xem, em giờ k thể miệt mài ngồi tìm tòi đc nữa   :D

Nhận xét một chút về tài liệu trên thì nếu bạn nào mới học hoặc cần học kĩ năng có thể đem nó ra tham khảo. Tuy nhiên khi viết một cái gì đấy thì không nên học tập mấy ông này, đại đa số là đem dịch từ tài liệu tiếng Anh sang và gần như chả có cái gì mới do công sức tìm tòi họ bỏ ra cả. Thậm chí bài viết của em trong đấy nó cũng như vậy, chỉ toàn lời giải cho bài toán và lời giải cho bài toán rất giống phong cách post bài của em từ xưa đến giờ ở diễn đàn. Nó chẳng có một chút ý nghĩa gì cả!




#564898 CMR: $\sum_{k=0}^n (-1)^k{m-1\choose k}{m\choose n-k}=(-1...

Đã gửi bởi Karl Heinrich Marx on 11-06-2015 - 07:44 trong Tổ hợp và rời rạc

Với $n,m$ là các số nguyên dương, chứng minh đẳng thức sau:

$$\Large \sum_{k=0}^n (-1)^k{m-1\choose k}{m\choose n-k}=(-1)^{\left\lfloor\frac{n}{2}\right\rfloor} {m-1\choose \left\lfloor\frac{n}{2}\right\rfloor}$$

Haha hôm qua em thấy topic của thầy rồi nhưng định bụng để đấy chưa làm để các bạn khác tham gia giải xem nhưng mà thấy thầy nhắc trên stt thôi thì vào hầu chuyện với thầy.

Vốn dĩ về mảng này em chỉ rành có một chiêu, gặp một cao thủ chuyên kĩ thuật biến đổi như thầy Thanh thì dẫu sao dùng một chiêu mà biến hóa đối đáp với thầy vài bài toán kể ra cũng là thành tựu rồi hehe.

Bây giờ ta xét đến vế trái là hệ số tự do của biểu thức:

$$\Large \sum_{k=0}^n (-1)^k{m-1\choose k}\frac{(1+x)^m}{x^{n-k}}$$

Chỗ này hơi khúc mắc là sigma chỉ chạy đến $n$ nó chả ăn nhập gì với cái chỉnh hợp cả, nếu thay $n$ bằng $m-1$ thì đẹp. Chú ý là nếu $k>m-1$ hoặc $k>n$ thì hệ số tự do của ${m-1\choose k}\frac{(1+x)^m}{x^{n-k}}$ dĩ nhiên bằng $0$. Điều này có nghĩa là hoàn toàn thay được $n$ bởi $m-1$ trong dấu sigma kia. Voilà, chìa khóa đây rồi!

Vế trái biểu thức ban đầu bằng hệ số tự do của khai triển:

$$\Large \sum_{k=0}^{m-1} (-1)^k{m-1\choose k}\frac{(1+x)^m}{x^{n-k}}=\frac{(1+x)^m}{x^n}\Large \sum_{k=0}^n (-1)^k{m-1\choose k}x^k=\frac{(1+x)^m(1-x)^{m-1}}{x^n}=\frac{(1+x)(1-x^2)^{m-1}}{x^n}$$

hệ số tự do trong biểu thức này chính là VP.




#563984 CMR: $\sum_{k=0}^n {k\choose a}{n-k...

Đã gửi bởi Karl Heinrich Marx on 06-06-2015 - 19:23 trong Tổ hợp và rời rạc

(Nhưng mà phân tích chỗ thừa này ra, lại chính là gợi ý cho lời giải của bài toán này! :)) Thế nên cho phép tôi Stop ở đây!)

 


Ở chỗ này theo thầy, biến đổi thế này
$\sum_{k_1+k_2\le n}{k_1\choose 2}=\sum_{k_1=2}^n\sum_{k_2=0}^{n-k_1}{k_1\choose 2}=\sum_{k=2}^n{k\choose 2}\sum_{k_2=0}^{n-k}1=\sum_{k=2}^n{k\choose 2}(n-k+1)$
Đỡ tốn công giải thích! :D

Mặt khác $\sum_{k=2}^n (n+1-k){k\choose 2}=\sum_{k=2}^n {n+1-k\choose 1}{k\choose 2}=\sum_{k+j=n+1} {j\choose 1}{k\choose 2}={n+2\choose 4}\;\;$ (Áp dụng luôn!)

Cảm ơn thầy đã giúp em chỉnh sửa phần biến đổi gọn gàng hơn!

Ok em đã hiểu vấn đề chỗ này rồi, vì cái sự lặp đó mà mình buộc phải thêm một bạn đánh dấu ở vị trí $k+1$ để dù có chọn $a$ bạn đầu trùng nhau thì vì cái vị trí chốt này mà giúp phân biệt bộ $a$ bạn chọn từ $k$ này khác cũng bộ $a$ bạn đó chọn từ $k$ khác. Thực ra nhìn vào đẳng thức thì cái việc thêm cái bạn ở vị trí chốt này là điều dễ dàng thấy được nhưng em cứ nghĩ chẳng hiểu thêm anh này vào làm gì và thêm một khúc mắc nữa là $a,b$ là những hằng số quên để ý VP làm e tưởng nhầm là tổng vế trái phụ thuộc vào mỗi cặp $a,b$  nên tự hỏi là khi chọn $a+b+1$ bạn bình đẳng như nhau thì tại sao lại chọn bạn chốt ngăn cách là đúng tại giữa $a$ và $b$ người, điều này không tự nhiên. Xem kĩ lại mới thấy thực ra trong từng $a+b+1$ người đấy chọn bất kì ai làm chốt cũng được và biểu thức vế trái chỉ phụ thuộc vào $a+b$ chứ không phụ thuộc vào cặp $(a,b)$.

Bài toán tổng quát thì có vẻ là tương tự. Nếu đây là một kết quả thầy tự mày mò ra thì thật là tuyệt vời! Nhưng kq nhìn đẹp thế này thì chắc chắn là có người tìm ra trước rồi :D Hehe điều đấy không quá quan trọng thầy nhể!




#563904 CMR: $\sum_{k=0}^n {k\choose a}{n-k...

Đã gửi bởi Karl Heinrich Marx on 06-06-2015 - 10:35 trong Tổ hợp và rời rạc

Bằng phương pháp đếm theo hai cách:

CMR: $\sum_{k=0}^n {k\choose a}{n-k\choose b}={n+1\choose a+b+1}$

 

Mở rộng:

 

$\sum_{k_1+k_2+...+k_m=n}{k_1\choose a_1}{k_2\choose a_2}...{k_m\choose a_m}={n+m-1\choose a_1+a_2+...+a_m+m-1}$

Em thấy khá thắc mắc là nếu ta nói số cách chọn $a+b$ bạn trong $n$ bạn là số bằng số cách chọn $a$ bạn từ $k$ bạn đầu tiên và $b$ bạn từ $n-k$ bạn còn lại thì sai cái gì nhỉ? Như vậy thành ra $\sum_{k=0}^n {k\choose a}{n-k\choose b}={n\choose a+b}$?

Thật sự xem những ứng dụng thầy Thanh post ở trên khá hay, một điều thú vị là các số $a_1,a_2,..,a_m$ có thể thay đổi một cách tùy ý miễn đảm bảo giữ nguyên tổng của chúng là được.

Mình có một vài ý thế này:

Nếu thêm một biến $k_{m+1}$ viết lại thế này:

$\sum_{k_1+k_2+...+k_m+k_{m+1}=n}{k_1\choose a_1}{k_2\choose a_2}...{k_m\choose a_m}{k_{m+1}\choose 0}={n+m\choose a_1+a_2+...+a_m+m}$

giờ lược biến $k_{m+1}$ này đi thì ta viết lại thế này:

$\sum_{k_1+k_2+...+k_m \le n}{k_1\choose a_1}{k_2\choose a_2}...{k_m\choose a_m}={n+m\choose a_1+a_2+...+a_m+m}$

Bây giờ cho $m=2,a_1=a_2=1$ thì ta có $\sum_{k_1+k_2 \le n}k_1k_2={n+2\choose 4}$

Hoặc cho $m=2,a_1=2,a_2=0$ thì:

$\sum_{k_1+k_2 \le n}{k_1\choose 2}={n+2\choose 4}$ hay là $\sum_{i=2}^n \sum_{k_1+k_2 =i}{k_1\choose 2}={n+2\choose 4}$

Chú ý là với mỗi số $k$ thì $k$ đóng $n-k+1$ lần vai trò là $k_1$ (với mỗi $i \ge k$ thì $k$ được xét làm $k_1$ một lần). Như vậy ta suy ra:

$\sum_{k=2}^n (n-k+1){k \choose 2} = {n+2\choose 4}$

Dùng kết quả ở trên của thầy Thanh thì ta có:

$(n+1)\sum_{k=2}^n{k \choose 2}=(n+1){n+1 \choose 3}=(n+2){n+1 \choose 3}-{n+1 \choose 3}=4 {n+2\choose 4}-{n+1 \choose 3}$

Vậy $\sum_{k=2}^n k{k \choose 2}=3 {n+2\choose 4}-{n+1 \choose 3}$

Với một đẳng thức linh hoạt thế này có thể điều chỉnh, gán một số giá trị ta sẽ cho ra được thêm nhiều đẳng thức thú vị khác!




#566550 CMR $\exists a,b,c:c^2\mid a^2+b^2$

Đã gửi bởi Karl Heinrich Marx on 18-06-2015 - 05:16 trong Số học

$\boxed{\text{Problem}}$

Chứng minh rằng $\forall n\in \mathbb{N}^*$ thì giữa $n^2$ và $(n+1)^2$ luôn tồn tại ba số tự nhiên phân biệt $a,b,c$ sao cho

$c^2\mid a^2+b^2$

Bài này a cũng có một chút ý tưởng nhưng cũng loay hoay một lúc k ra, post lên để các e thử tiếp tục xem sao (dĩ nhiên là khi không ra thì vẫn hơi nghi ngờ tính đúng đắn của bài toán). Ta thử giải quyết với $n$ vô cùng lớn trước xem sao, đầu tiên ta thấy xét $\frac{a^2+b^2}{c^2}$ trong đó $n^2 \le a,b,c, \le (n+1)^2$ có thể suy ra là:

$$ \frac{2n^2}{(n+1)^2} \le \frac{a^2+b^2}{c^2} \le \frac{2(n+1)^2}{n^2}$$

Cho $n$ vô cùng lớn thì $\frac{a^2+b^2}{c^2} \rightarrow 2$.

Vậy với $n$ đủ lớn mà tồn tại $3$ số thỏa mãn đề bài thì ta phải có $a^2+b^2=2c^2 \Rightarrow (a-c)(a+c)=(c-b)(c+b)$

Đặt $(a-c)=k(c-b)$ khi đó $k \in Q$ và suy ra $b+c=k(a+c)$. Từ $2$ pt này có thể giải $a,b$ theo $k$ và $c$. Có vẻ như sẽ ra $a= \frac{1-k^2+2k}{k^2+1}.c,b=\frac{k^2-1+2k}{k^2+1}.c$. Đặt $k=\frac{u}{v}$ trong đó $u,v$ là các số nguyên, ta biểu diễn được $\frac{a}{c},\frac{b}{c}$ theo các phân thức với $u,v$. Bây giờ phải chứng minh tồn tại $u,v,$ để từ các phân thức đó nằm trong đoạn $(n^2,(n+1)^2$. Tuy nhiên chỗ này a thử cho một vài giá trị $u,v$ theo $n$ thì không ra. Chú ý một chút là vì giới hạn của $\frac{a}{c},\frac{b}{c}$ đều bằng $1$ khi $n$ về vô cùng nên khi biểu diễn $u,v$ theo $n$ nên chọn giá trị sao cho giới hạn của $\frac{u}{v}$ khi $n$ về vô cùng là $1$. Nếu e có lời giải bài này thì a rất muốn xem vì theo anh thì hình như nó không đúng với $n$ đủ lớn.




#566707 CMR $\exists a,b,c:c^2\mid a^2+b^2$

Đã gửi bởi Karl Heinrich Marx on 18-06-2015 - 18:37 trong Số học

em không có lời giải của bài toán cũng như không dám chắc về tính đúng đắn của nó

theo em thì cần một điều kiện gì đó của $n$ đủ chặt để bài toán đúng ,mong anh và mọi người góp ý

Khi thay $k= \frac{u}{v}$ thì ta có $\frac{a}{c}=\frac{u^2-v^2+2uv}{u^2+v^2}=1+\frac{2v(u-v)}{u^2+v^2},\frac{b}{c}=\frac{v^2-u^2+2uv}{u^2+v^2}=1+\frac{2u(v-u)}{u^2+v^2}$

Giờ nếu thay $u=n+2,v=n$ thì sẽ ra $\frac{a}{c}=\frac{n^2+4n+2}{n^2+2n+2},\frac{b}{c}=\frac{n^2-2}{n^2+2n+2}$

Trong ví dụ này nếu lấy đoạn $n^2;(n+2)^2$ cũng không được nhưng nếu lấy $n^2-2;(n+2)^2-2$ thì lại hợp lí. Tuy nhiên lời giải ta lại sử dụng đúng 2 mút thì nó lại mất hay vì thường người ta sẽ thử 2 mút trước ra ngay đáp án thì họ chả quan tâm phần làm sao tìm ra 2 mút này cả.

Có một cách tìm cận không chặt lắm nhưng mà tự nhiên hơn là thay số như ở trên.

Ta hình dung là $c$ sẽ nằm giữa 2 số $a,b$ và $|\frac{2v(u-v)}{u^2+v^2}|,|\frac{2u(v-u)}{u^2+v^2}|$ là các khảng cách từ $a,b$ đến $c$. Khoảng cách này càng bé càng tốt mà $u,v$ không thể bằng nhau nên ta cho $v=u+1$. 

Thay vào thì ta được $\frac{a}{c}=\frac{2v^2+4v+1}{2v^2+2v+1}, \frac{b}{c}=\frac{2v^2-1}{2v^2+2v+1}$

Chọn một cận là $n^2$ cận còn lại là $n^2+t$, trong 2 phân thức trên số ở mẫu nằm giữa 2 số ở tử nên chỉ cần chặn 2 số ở tử thôi.

tức là $2v^2-1 \ge n^2, 2(v+1)^2 \le n^2+t+1$

Tìm $t$ để thỏa mãn tồn tại số nguyên $v$. Cái này các e thử tiếp tục xem.

Việc đặt ra vấn đề chưa giải quyết đc và trao đổi với nhau như thế này là điều cần thiết khi tham gia diễn đàn, các e cố gắng phát huy nhé!




#562038 Chứng minh: $\sum_{k=0}^n \left(\frac{-1...

Đã gửi bởi Karl Heinrich Marx on 28-05-2015 - 00:40 trong Tổ hợp và rời rạc

Chứng minh đẳng thức sau:

 

$\sum_{k=0}^n \left(\frac{-1}{2}\right)^k{n\choose k}{2k+1\choose k}= \dfrac{(-1)^n\left(\frac{2n-1-(-1)^n}{2}\right)!!}{\left(\frac{2n+1-(-1)^n}{2}\right)!!}$

 

Trong đó: $\begin{cases}(2m)!!=2^m.m!\\ (2m-1)!!=\dfrac{(2m)!}{2^m.m!}\end{cases}$

Bài này có thể sử dụng kĩ thuật tính hệ số đa thức khá hiệu quả. Tổng cần tính chính là hệ số tự do trong khai triển:

$$ \sum_{k=0}^n \left(\frac{-1}{2}\right)^k{n\choose k} \frac{(x+1)^{2k+1}}{x^k}$$

$$= (x+1) \sum_{k=0}^n {n\choose k}\frac{(-1)^k(x+1)^{2k}}{(2x)^k} = (x+1)\left( 1-\frac{(x+1)^2}{2x} \right)^n=(x+1)\frac{(-1)^n(x^2+1)^n}{2^nx^n}$$

Hệ số tự do của khai triển này chính bằng $$\frac{(-1)^n}{2^n}{n\choose \lfloor \frac{n}{2} \rfloor}$$

Có thể kiểm tra cái này bằng với vế phải, e k biết có phải thầy biến đổi từ giá trị này ra công thức tường minh theo n không có dấu giá trị tuyệt đối không? Nếu là như vậy thì em rất muốn biết thầy biến đổi ntn :D

E không hiểu sao nó k hiện công thức dù đã cố gắng sửa rồi, hi vọng thầy đọc và hiểu được. Em rất thích đọc những bài toán do thầy post vì trước khi post 1 bài toán thầy đều đầu tư ít nhiều vào đó!




#563156 Chứng minh rằng tồn tại 2 nhà phê bình bỏ phiếu như nhau

Đã gửi bởi Karl Heinrich Marx on 03-06-2015 - 06:14 trong Tổ hợp và rời rạc

 

Có 3366 nhà phê bình điện ảnh bỏ phiếu cho giải Oscar. Mỗi một nhà phê bình bỏ phiếu cho đúng 1 nam diễn viên và đúng 1 nữ diễn viên. Sau khi các nhà phê bình bỏ phiếu xong, người ta thấy rằng với mỗi n thuộc {1, 2, ..., 100} đều có một nam diễn viên hoặc 1 nữ diễn viên được đúng n phiếu. Chứng minh rằng tồn tại 2 nhà phê bình bỏ phiếu như nhau (tức là bỏ phiếu cho cùng 1 nam diễn viên và 1 nữ diễn viên).

(Balkan MO 2015)

 

Trước tiên ta khai thác một số điều từ giả thiết. Tổng số phiếu sẽ là $\frac{100.101}{2}=5050$, mà chỉ có $3366$ nhà phê bình nên phải có ít nhất $5050-3366=1684$ người bỏ 2 phiếu. Rõ ràng từ câu hỏi của bài toán thì ta phải xem xét đến $1684$ ông này, để chứng minh có $2$ ông bỏ phiếu giống nhau thì chỉ có thể rơi vào $1684$ vị này là khả thi thôi. Tức là nếu xét các cặp được cùng một nhà phê bình bỏ phiếu thì trong $100$ diễn viên đề cập trong đề phải có ít nhất $1684$ cặp nam nữ. Đến đây ý tưởng sẽ phải chứng minh không thể có đến $1684$ cặp được. Bây giờ ta sẽ cố gắng tạo ra nhiều cặp nam nữ nhất trong số $100$ người này

Không mất tính tổng quát giả sử có $k$ người nam và $100-k$ người nữ trong đó $k \le 50$. Như vậy nhiều nhất ta tạo ra được $(100-k)k$ cặp. Tuy nhiên như thế thì $k$ nam sẽ có $100-k$ phiếu và $100-k$ nữ sẽ có $k$ phiếu. Như vậy số phiếu của những người này đều không nhỏ hơn $k$, điều này không hợp lí. Buộc ta phải bỏ bớt các cặp trong này để đưa một số giá trị về $1,2,3,..,k-1$, chú ý là $100-k \ge k$ nên để loại bỏ ít cặp nhất ta sẽ chọn ra $k-1$ cô gái trong $100-k$ người có $k$ phiếu và bỏ bớt phiếu những người này tạo thành dãy $1,2,..,k-1$. Như vậy ta đã bỏ đi $k.(k-1)-(1+2+..+k-1)=\frac{k(k-1)}{2}$. Vậy chỉ có thể tạo ra nhiều nhất là $(100-k)k-\frac{k(k-1)}{2}=k(100-k-\frac{k-1}{2})$. Áp dụng Cauchy thì ta có:

$$ \frac{3}{2}k(100-k-\frac{k-1}{2}) \le \frac{1}{4}(100+1/2)^2<\frac{100.101}{4} \Rightarrow k(100-k-\frac{k-1}{2}) < \frac{100.101}{6}=\frac{5050}{3}<1684$$

Vậy là kết quả được chứng minh.

Dù là giải đi đến kết quả thậm chí có thể thấy là ta hoàn toàn có thể tổng quát $100$ thành $n$ và thay đổi số nhà phê bình bé hơn $\frac{n(n+1)}{3}$. Tuy nhiên chúng ta cần nhìn nhận vấn đề cẩn thận hơn một chút, việc chứng minh này chưa có nhiều ý nghĩa lắm. Mình sẽ nêu ra một số vấn đề để các bạn tìm hiểu để hiểu rõ bài toán hơn:

1. Đầu tiên hãy xem khi tiếp cận bài toán các bạn suy nghĩ ntn? Dĩ nhiên các bạn sẽ phải hình dung đến một cấu hình tốt nhất nào đấy để cố gắng tìm điểm mâu thuẫn ở đó.

2.. Nếu ta tăng số nhà phê bình thêm 1 hoặc 2 đơn vị. Khi đó đpcm không đúng nữa, vậy cấu hình giám khảo chấm như thế nào để nó không đúng (đây có thể gọi là cấu hình "tốt nhất", dĩ nhiên hướng xây dựng cấu hình này sẽ dựa vào dấu bất đẳng thức ở trên). Từ đó các bạn có thể thấy được cách đưa bài toán vào trạng thái tối ưu.

3. Có một vấn đề là ta mới chỉ ra mâu thuẫn ở việc đánh giá của 1684 giám khảo chấm cả 2 phiếu trên 100 người thôi. Giả sử ta đáp ứng được số cặp trong 100 người thỏa mãn bđt trên, liệu những người chỉ bỏ 1 phiếu còn lại có thể hoàn thành được đk của bài toán là cứ $1 \le i \le 100$ thì có một người có $i$ phiếu? Thay $100$ bằng $n$ thì sẽ rơi vào những vấn đề gì? Đổi lại tìm số giám khảo nhiều nhất để đảm bảo có 2 người bỏ phiếu giống nhau thì xử lí thế nào với $n$?

4. Nếu vấn đề đòi hỏi đến 2 cặp giám khảo bỏ phiếu giống nhau thì thế nào? hoặc một giám khảo có thể bỏ phiếu cho 2 người nam và 1 người nữ thì ntn?

Khi tìm hiểu bài toán đủ nhiều các bạn có thể tự đặt ra thêm những câu hỏi và tự giải quyết chúng. Đôi khi có thể đặt ra rồi không giải quyết được nhưng chắc chắn là không vô ích!

Hi vọng các bạn tham gia thảo luận nhiều hơn là đưa ra một lời giải.

P/s em chủ topic: À muốn hỏi e một vấn đề nhỏ, thầy Thông trong chữ kí của em có phải là thầy Nguyễn Văn Thông ở LQĐ Đà Nẵng k nhỉ?




#722206 Chuyên đề Đẳng thức Tổ hợp

Đã gửi bởi Karl Heinrich Marx on 14-05-2019 - 02:50 trong Tài nguyên Olympic toán

Lâu lắm rồi không "ngó" đến chủ đề này. Qua một khoảng thời gian tương đối lâu tôi thử tìm kiếm trên mạng về các bài viết, tài liệu hướng dẫn, luyện thi, vân vân... liên quan đến các đẳng thức hệ số tổ hợp thì chợt nhận ra rằng: Hầu hết các tài liệu đó đều mang tính khuôn mẫu, na ná như nhau và rất nhàm chán!
Chẳng hạn như: "Ứng dụng đạo hàm và tích phân để chứng minh và tính tổng các hệ số nhị thức"
Theo tôi thấy nó chỉ "tiện" chứ không nêu lên được bản chất nội tại của nó như quy tắc hút hay đảo chiều, v.v...
Hay như việc nhận biết và đưa vào một số thủ thuật nhỏ trong "dấu hiệu" của tổng (viết dưới dạng liệt kê) chỉ đơn thuần là các phép biến đổi cơ bản khi viết tổng dưới dạng $\sum$
Rất nhiều tài liệu, phương pháp trên mạng không thể giải quyết nổi dù là một bài tập trong ĐTTH ở đây bạn có tin không?
Không bàn đến "độ khó" mà là phương thức tiếp cận một bài toán cần phải "linh động" hơn, đôi khi chẳng cần phải đạo hàm, tích phân hay biến đổi gì đó mà chỉ cần tìm cách "đếm" là đủ!
ĐTTH đã tồn tại một thời gian khá lâu mà chưa có một cuốn sách nào một tài liệu (tiếng Việt) tương đương nào xứng "tầm" với nó, quả thực là một điều đáng buồn!
Đôi lời cảm nhận cá nhân, bạn thấy thế nào? (Gạch đá quăng hết vào đây :D)

 

Đến hôm nay em mới được đọc tài liệu này của mọi người (rất tiếc vì tài liêu này ra vào thời điểm mà em còn không quan tâm đến toán nhiều nữa). Phải công nhận là tài liệu này rất hay và có một sự đầu tư lớn. Tuy nhiên em có một số ý kiến cá nhân như thế này.

 

Tài liệu này thực sự thiên quá nhiều về kĩ thuật. Ở một khía cạnh nào đó, đây là một tài liệu xuất sắc trong việc đưa ra những kĩ thuật chứng minh đẳng thức. Theo ý kiến chủ quan của em thì có lẽ chỉ một số ít các bạn thi QG hay gần như thế là có thể đọc hết được.

 

Tài liệu trình bày rất tốt cách chứng minh các đẳng thức, tuy nhiên không nhiều trong đó nêu ra những đẳng thức đấy thể hiện điều gì và tại sao người ta đặt ra được một đẳng thức như vậy. Tài liệu này có thể giúp chúng ta thỏa mãn việc giải được toán chứ chưa hẳn là hiểu được toán.

 

Cái hay của toán học đó là nó trình bày thực tế bằng ngôn ngữ logic (vì thế nên nó khó hiểu). Chẳng hạn để thể hiện hàm số $f$ có đồ thị là một đường liền nét, vậy trong ngôn ngữ toán liền nét định nghĩa như thế nào? Tại mọi điểm $x_0$ thì hàm $f$ tồn tại lim trái và lim phải khi $x$ tiến đến $x_0$ và 2 giá trị này đều bằng $f(x_0)$, hay là chúng ta hỏi một đứa nhỏ hình tròn là hình gì? Dù rằng hẳn là nó sẽ biết hình tròn như thế nào và hình dung được nhưng khẩ năng cao là nó không thể đưa ra cái định nghĩa là tập hợp tất cả những điểm cách đều một điểm cố định một khoảng cách không đổi được :)). Có thể không nhất thiết chúng ta đưa ra cách hình dung cụ thể cho những công thức toán nhưng có thể chỉ cho người đọc thấy rằng một công thức hay một bài toán trông khó như vậy, nó được xây dựng lên từ những điều cơ sở nào, bằng những cách tương tự như thế chúng ta có thể tạo ra những bài toán khó khác như thế nào? Em nghĩ những điều như thế cần cho cộng đồng các bạn học toán hơn, nó sẽ giúp nhiều bạn yêu thích toán hơn.

 

Điều cuối cùng em chỉ muốn nói là em chém gió vậy thôi chứ bảo em viết một tài liệu như em nói thì em cũng không làm được đâu :)) Gạch đá em xin nhận :))

Chúc mừng mọi người vì đã tạo ra một tài liệu tuyệt vời. Nếu như mọi người có một dự án hay ho gì đó sắp tới thì hi vọng em có thể đóng góp chút gì đó.




#564899 Cho dãy $u_n=\sum_{k=0}^{2n} \frac{1...

Đã gửi bởi Karl Heinrich Marx on 11-06-2015 - 08:22 trong Dãy số - Giới hạn

Cho dãy số: $\qquad u_n=\sum_{k=0}^{2n} \frac{1}{n^2+k}\qquad (n=1,2,...)$

 

Tính tổng: $ \quad S_n=\sum_{k=1}^n \left\lfloor\frac{2}{u_k}\right\rfloor $

Bài toán mới nhìn vào có vẻ hơi đáng sợ nhưng bình tĩnh một chút đánh giá khách quan thì kiểu toán này chỉ có cách chặn 2 đầu thôi.

Ta có:

$$ \sum_{k=0}^{2n}\frac{1}{n^2}>u_n=\sum_{k=0}^{2n} \frac{1}{n^2+k}>\sum_{k=0}^{2n}\frac{1}{(n+1)^2} \Rightarrow \frac{2n+1}{n^2}>u_n>\frac{2n+1}{(n+1)^2}$$

Tuy nhiên đến đây mà chia 2 lật ngược lại thì chưa đạt được điều ta mong muốn, bây giờ ta cần một điều chặt hơn một chút là

$$ \frac{2n}{n^2}>u_n>\frac{2n+2}{(n+1)^2}$$

Tức là ta cần một tổng $i+1$ số hạng trong sigma biểu diễn $u_n$ có tổng bé hơn $\frac{i}{n^2}$ tức là $i+1$ số này đều không nhỏ hơn $\frac{i+1}{i}.n^2$, tự nhiên ta thử với $i=n$ thì đúng luôn chọn các số từ $n(n+1)$ đến $n(n+2)$. Như vậy ta viết lại $u_n$

$$ u_n=\sum_{k=0}^{n-1}\frac{1}{n^2+k}+\sum_{k=n}^{2n}\frac{1}{n^2+k}<n.\frac{1}{n^2}+(n+1).\frac{1}{n(n+1)}=\frac{2}{n}$$

$$ u_n=\sum_{k=0}^{n-1}\frac{1}{n^2+k}+\sum_{k=n}^{2n}\frac{1}{n^2+k}>n.\frac{1}{n(n+1)}+(n+1).\frac{1}{(n+1)^2}=\frac{2}{n+1}$$

Do vậy $n<\frac{2}{u_n}<n+1$

Suy ra $S_n=\frac{n(n+1)}{2}$